Minimum Normalizer












0












$begingroup$


Let $x_1ge x_2gedots ge x_n> 0$. What is the minimum value of $k$ such that $$frac{nkx_i}{x_1+dots+x_n}ge 1quad forall i.$$



I tried a bit hoping that $k=1+frac{1}{2}+dots+frac{1}{n}$ would work. But not getting any clue. Any hint is greatly appreciated.










share|cite|improve this question









$endgroup$

















    0












    $begingroup$


    Let $x_1ge x_2gedots ge x_n> 0$. What is the minimum value of $k$ such that $$frac{nkx_i}{x_1+dots+x_n}ge 1quad forall i.$$



    I tried a bit hoping that $k=1+frac{1}{2}+dots+frac{1}{n}$ would work. But not getting any clue. Any hint is greatly appreciated.










    share|cite|improve this question









    $endgroup$















      0












      0








      0





      $begingroup$


      Let $x_1ge x_2gedots ge x_n> 0$. What is the minimum value of $k$ such that $$frac{nkx_i}{x_1+dots+x_n}ge 1quad forall i.$$



      I tried a bit hoping that $k=1+frac{1}{2}+dots+frac{1}{n}$ would work. But not getting any clue. Any hint is greatly appreciated.










      share|cite|improve this question









      $endgroup$




      Let $x_1ge x_2gedots ge x_n> 0$. What is the minimum value of $k$ such that $$frac{nkx_i}{x_1+dots+x_n}ge 1quad forall i.$$



      I tried a bit hoping that $k=1+frac{1}{2}+dots+frac{1}{n}$ would work. But not getting any clue. Any hint is greatly appreciated.







      calculus






      share|cite|improve this question













      share|cite|improve this question











      share|cite|improve this question




      share|cite|improve this question










      asked Jan 13 at 12:06









      KumaraKumara

      221118




      221118






















          1 Answer
          1






          active

          oldest

          votes


















          1












          $begingroup$

          You want to find the minimum $k>0$ such that
          $$
          k, x_igeq overline{x}, quad forall i = 1, ldots, n,
          qquad
          text{where} overline{x} := frac{x_1 + cdots + x_n}{n},.
          $$

          Since, by assumption, $x_1 geq x_2 geq cdots geq x_n > 0$, the above $n$ inequalities are satisfied if and only if $k, x_n geq overline{x}$. Hence, the minimum of such values of $k$ is $k = overline{x} / x_n$.






          share|cite|improve this answer











          $endgroup$













          • $begingroup$
            Sure, you are right, for some reason I was considering the reversed inequality!
            $endgroup$
            – Rigel
            Jan 13 at 12:49










          • $begingroup$
            Thanks. That's right. However, is it possible to obtain the bound in terms of $k=1+frac{1}{2}+dots+frac{1}{n}$ possibly with some multiplicative (constant) factor?
            $endgroup$
            – Kumara
            Jan 13 at 15:46













          Your Answer





          StackExchange.ifUsing("editor", function () {
          return StackExchange.using("mathjaxEditing", function () {
          StackExchange.MarkdownEditor.creationCallbacks.add(function (editor, postfix) {
          StackExchange.mathjaxEditing.prepareWmdForMathJax(editor, postfix, [["$", "$"], ["\\(","\\)"]]);
          });
          });
          }, "mathjax-editing");

          StackExchange.ready(function() {
          var channelOptions = {
          tags: "".split(" "),
          id: "69"
          };
          initTagRenderer("".split(" "), "".split(" "), channelOptions);

          StackExchange.using("externalEditor", function() {
          // Have to fire editor after snippets, if snippets enabled
          if (StackExchange.settings.snippets.snippetsEnabled) {
          StackExchange.using("snippets", function() {
          createEditor();
          });
          }
          else {
          createEditor();
          }
          });

          function createEditor() {
          StackExchange.prepareEditor({
          heartbeatType: 'answer',
          autoActivateHeartbeat: false,
          convertImagesToLinks: true,
          noModals: true,
          showLowRepImageUploadWarning: true,
          reputationToPostImages: 10,
          bindNavPrevention: true,
          postfix: "",
          imageUploader: {
          brandingHtml: "Powered by u003ca class="icon-imgur-white" href="https://imgur.com/"u003eu003c/au003e",
          contentPolicyHtml: "User contributions licensed under u003ca href="https://creativecommons.org/licenses/by-sa/3.0/"u003ecc by-sa 3.0 with attribution requiredu003c/au003e u003ca href="https://stackoverflow.com/legal/content-policy"u003e(content policy)u003c/au003e",
          allowUrls: true
          },
          noCode: true, onDemand: true,
          discardSelector: ".discard-answer"
          ,immediatelyShowMarkdownHelp:true
          });


          }
          });














          draft saved

          draft discarded


















          StackExchange.ready(
          function () {
          StackExchange.openid.initPostLogin('.new-post-login', 'https%3a%2f%2fmath.stackexchange.com%2fquestions%2f3071942%2fminimum-normalizer%23new-answer', 'question_page');
          }
          );

          Post as a guest















          Required, but never shown

























          1 Answer
          1






          active

          oldest

          votes








          1 Answer
          1






          active

          oldest

          votes









          active

          oldest

          votes






          active

          oldest

          votes









          1












          $begingroup$

          You want to find the minimum $k>0$ such that
          $$
          k, x_igeq overline{x}, quad forall i = 1, ldots, n,
          qquad
          text{where} overline{x} := frac{x_1 + cdots + x_n}{n},.
          $$

          Since, by assumption, $x_1 geq x_2 geq cdots geq x_n > 0$, the above $n$ inequalities are satisfied if and only if $k, x_n geq overline{x}$. Hence, the minimum of such values of $k$ is $k = overline{x} / x_n$.






          share|cite|improve this answer











          $endgroup$













          • $begingroup$
            Sure, you are right, for some reason I was considering the reversed inequality!
            $endgroup$
            – Rigel
            Jan 13 at 12:49










          • $begingroup$
            Thanks. That's right. However, is it possible to obtain the bound in terms of $k=1+frac{1}{2}+dots+frac{1}{n}$ possibly with some multiplicative (constant) factor?
            $endgroup$
            – Kumara
            Jan 13 at 15:46


















          1












          $begingroup$

          You want to find the minimum $k>0$ such that
          $$
          k, x_igeq overline{x}, quad forall i = 1, ldots, n,
          qquad
          text{where} overline{x} := frac{x_1 + cdots + x_n}{n},.
          $$

          Since, by assumption, $x_1 geq x_2 geq cdots geq x_n > 0$, the above $n$ inequalities are satisfied if and only if $k, x_n geq overline{x}$. Hence, the minimum of such values of $k$ is $k = overline{x} / x_n$.






          share|cite|improve this answer











          $endgroup$













          • $begingroup$
            Sure, you are right, for some reason I was considering the reversed inequality!
            $endgroup$
            – Rigel
            Jan 13 at 12:49










          • $begingroup$
            Thanks. That's right. However, is it possible to obtain the bound in terms of $k=1+frac{1}{2}+dots+frac{1}{n}$ possibly with some multiplicative (constant) factor?
            $endgroup$
            – Kumara
            Jan 13 at 15:46
















          1












          1








          1





          $begingroup$

          You want to find the minimum $k>0$ such that
          $$
          k, x_igeq overline{x}, quad forall i = 1, ldots, n,
          qquad
          text{where} overline{x} := frac{x_1 + cdots + x_n}{n},.
          $$

          Since, by assumption, $x_1 geq x_2 geq cdots geq x_n > 0$, the above $n$ inequalities are satisfied if and only if $k, x_n geq overline{x}$. Hence, the minimum of such values of $k$ is $k = overline{x} / x_n$.






          share|cite|improve this answer











          $endgroup$



          You want to find the minimum $k>0$ such that
          $$
          k, x_igeq overline{x}, quad forall i = 1, ldots, n,
          qquad
          text{where} overline{x} := frac{x_1 + cdots + x_n}{n},.
          $$

          Since, by assumption, $x_1 geq x_2 geq cdots geq x_n > 0$, the above $n$ inequalities are satisfied if and only if $k, x_n geq overline{x}$. Hence, the minimum of such values of $k$ is $k = overline{x} / x_n$.







          share|cite|improve this answer














          share|cite|improve this answer



          share|cite|improve this answer








          edited Jan 13 at 12:48

























          answered Jan 13 at 12:25









          RigelRigel

          11.2k11320




          11.2k11320












          • $begingroup$
            Sure, you are right, for some reason I was considering the reversed inequality!
            $endgroup$
            – Rigel
            Jan 13 at 12:49










          • $begingroup$
            Thanks. That's right. However, is it possible to obtain the bound in terms of $k=1+frac{1}{2}+dots+frac{1}{n}$ possibly with some multiplicative (constant) factor?
            $endgroup$
            – Kumara
            Jan 13 at 15:46




















          • $begingroup$
            Sure, you are right, for some reason I was considering the reversed inequality!
            $endgroup$
            – Rigel
            Jan 13 at 12:49










          • $begingroup$
            Thanks. That's right. However, is it possible to obtain the bound in terms of $k=1+frac{1}{2}+dots+frac{1}{n}$ possibly with some multiplicative (constant) factor?
            $endgroup$
            – Kumara
            Jan 13 at 15:46


















          $begingroup$
          Sure, you are right, for some reason I was considering the reversed inequality!
          $endgroup$
          – Rigel
          Jan 13 at 12:49




          $begingroup$
          Sure, you are right, for some reason I was considering the reversed inequality!
          $endgroup$
          – Rigel
          Jan 13 at 12:49












          $begingroup$
          Thanks. That's right. However, is it possible to obtain the bound in terms of $k=1+frac{1}{2}+dots+frac{1}{n}$ possibly with some multiplicative (constant) factor?
          $endgroup$
          – Kumara
          Jan 13 at 15:46






          $begingroup$
          Thanks. That's right. However, is it possible to obtain the bound in terms of $k=1+frac{1}{2}+dots+frac{1}{n}$ possibly with some multiplicative (constant) factor?
          $endgroup$
          – Kumara
          Jan 13 at 15:46




















          draft saved

          draft discarded




















































          Thanks for contributing an answer to Mathematics Stack Exchange!


          • Please be sure to answer the question. Provide details and share your research!

          But avoid



          • Asking for help, clarification, or responding to other answers.

          • Making statements based on opinion; back them up with references or personal experience.


          Use MathJax to format equations. MathJax reference.


          To learn more, see our tips on writing great answers.




          draft saved


          draft discarded














          StackExchange.ready(
          function () {
          StackExchange.openid.initPostLogin('.new-post-login', 'https%3a%2f%2fmath.stackexchange.com%2fquestions%2f3071942%2fminimum-normalizer%23new-answer', 'question_page');
          }
          );

          Post as a guest















          Required, but never shown





















































          Required, but never shown














          Required, but never shown












          Required, but never shown







          Required, but never shown

































          Required, but never shown














          Required, but never shown












          Required, but never shown







          Required, but never shown







          Popular posts from this blog

          Can a sorcerer learn a 5th-level spell early by creating spell slots using the Font of Magic feature?

          ts Property 'filter' does not exist on type '{}'

          Notepad++ export/extract a list of installed plugins